Sie sind auf Seite 1von 5

needless presentation or cumulative issues.

” (c) Rationale:
✔ General Overview 3) Relevance Exception: Character Evidence/Habit Evidence (i) Encourage victims to step forward
a) No specific FRE governs similar happenings evidence, but the Simon court stated
that such evidence is inadmissible unless admission is allowed under Rule 403 , and "the
a) Hearsay Note: If hearsay of reputation as to character, look to 803(21) – “"The (ii) Protect privacy w/ victim similar happening occurred under circumstances ‘substantially similar’ to those
1) Evidence in General following are not excluded by the hearsay rule, even though the declarant is available as a (iii) Protect risk of jury misusing the evid (e.g. forceable sex less deserving surrounding the event in question.”
a) Evidence defined: The material from which inferences may be drawn as the basis witness… Reputation of a person’s character among associates or in the community” of punishment for promiscuous women) b) Admissibility of Prior Accidents: Where similar accidents were caused by the
for proof of the truth or falsity of a disputed fact b) INITIAL ISSUE: First, we must determine whether the evidence at issue is character (iv) Prohibits a propensity inference b/c of past conduct same event or condition, evidence of those prior accidents or injuries is admissible to
b) Types of Evidence: evidence, or habit evidence. (4) EXCEPTIONS: FRE 412(b)(1) “In a criminal case, the following evid is prove:
i) Direct evidence – i) Character refers to a person's general propensity or disposition in respect to a admissible, if otherwise admissible under these rules: i) That a defect or a dangerous condition existed
(1) evidence that proves a proposition without relying on any inference (e.g., general trait (e.g. honesty, violence, cowardice, carefulness) (a) Not source of Semen/Injury:(A) evid of specific instances of sexual ii) That defendant had knowledge of the defect or dangerous condition
witness testimony) behavior by the alleged victim offered to prove that a person other than the iii) That the defect or dangerous condition was the cause of the present injury
ii) Exam: Now, state that “the evidence could be character, but could not be. We’ll
(2) Almost always admissible accused was the source of semen, injury, or other physical evidence; 5) Relevance Exception: Subsequent Precaution:
now analyze character before we get to the issue of whether it is habit evidence.”
ii) Indirect/Circumstantial evidence – (b) Past Consent(B) evid of specific instances of sexual behavior by the a) Language FRE 407: “When, after an injury or harm allegedly caused by an event,
c) FIRST ISSUE – IS CHARACTER EVIDENCE BEING OFFERED FOR PROPER
(1) evidence of a subsidiary fact from which the existence of an ultimate fact may alleged victim w/r/t the person accused of the sexual misconduct offered by the measures are taken that, if taken previously, would have made the injury or harm less
PURPOSE:
be inferred accused to prove consent or by the prosecution; and likely to occur, evid of the subsequent measures is not admissible to prove negligence,
(2) More often subject to the exclusionary rules of evidence and relevancy i) Initial Note: If it is an essential element of the claim or defense, then 404 doesn’t
exclude it. Another way to look at this is, is this “in issue?” (c) Constitutional Rights(C) evid the exclusion of which would violate the culpable conduct, a defect in a product, a defect in a product’s design, or a need for a
c) 3 Basic forms of evidence constitutional rights of the Δ” warning or instruction.
i) Testimonial evidence – Oral testimony given under oath by a witness in court or ii) Admissibility/inadmissibility Language 404(a): “Evidence of a person’s
character or a trait of character is not admissible for the purpose of proving action in (5) EXCEPTIONS: FRE 412(b)(2): “In a civil case: b) Rationale: Encourage people to take further precautionary safety measures
in a pretrial deposition (a) Evidence offered to prove the sexual behavior or sexual predisposition of c) Allowable, b/c not in above rule: a proponent may offer evidence of subsequent
conformity therewith on a particular occasion, except:
ii) Tangible Evidence – evidence in the form of exhibits and is either real or any alleged victim is admissible if it is otherwise admissible under these rules measures when offered for proving:
(1) Note on “In Conformity Therewith”:
demonstrative and its probative value substantially outweighs the danger of harm to any victim i) Ownership, control, impeachment
(a) Explanation: Can't show the person is bad or that she had propensity to
(1) Real – the real thing at issue in the case (contract, murder weapon) and of unfair prejudice to any party. ii) OR feasibility of making the change IF the opponent argues that the subsequent
commit the crime charged. If you have to infer something about a person’s
(2) Demonstrative – not the real item involved in the case, rather a visual or character, then it is improper. Pattern of bad conduct is inadmissible, it is (b) Evidence of an alleged victim's reputation is admissible only if it has been measure was not feasible.
audiovisual aid for the fact finder. "propensity" placed in controversy by the alleged victim. 6) Relevance Exception: Offers for Compromise
(a) Must be a “fair” representation v) SEX CRIMES – Sexual Assault: FRE 413 a) Paraphrased FRE 408: Settlement offers and agreements and statements made
(b) Rationale: Relevancy is outweighed by the risk of prejudice and
iii) Tangible-Testimonial Evidence – Hybrid of the above two forms (Transcribed confusion of issues (1) General Rule: Basically, In sexual assault cases, evidence that defendant during such compromise negotiations are not admissible to show liability or non-liability for
deposition) committed past offenses of sexual assault is admissible for any relevant matter, an underlying claim, nor its validity or non-validity.
(2) EXCEPTION (1) Character of accused—Evid of a pertinent trait of character
iv) Judicial Notice (Distinguishable) – this is a matter that, because it is common
offered by an accused, or by the prosecution to rebut the same, or if evid of a trait doesn’t matter if conviction was had. b) Exceptions:
knowledge or may be readily verified, need not be proven. (i.e., don’t need to offer of character of the alleged victim of the crime is offered by the accused and (2) Notice: If the government intends to produce such evidence, it needs to i) Otherwise admissible evidence is not deemed inadmissible merely because it is
evidence.) admitted under Rule 404(a)(2), [then] evid of the same trait of character of the disclose that evidence to the defendant 15 days before trial, or at a later time if the presented in the course of compromise negotiations.
2) Procedure for Admitting or Excluding Evidence accused offered by the prosecution; court allows it ii) Evidence is admissible if offered for another purpose besides culpability (bias,
a) Dynamic at Trial: Manner in which evidence issues are raised at trial: proponent (3) EXCEPTION (2) Character of the alleged victim—Evid of the pertinent trait of (3) Definition: The rule defines “offense of sexual assault” in subsection (d) prejudice of a witness, proving an effort to obstruct a criminal investigation or
offers evidence and opponent puts up the obstacles by objecting to the evidence. character of the alleged victim of the crime offered by the accused, or by the (a) any conduct proscribed by chapter 109A of title 18, USC; prosecution)
b) Grounds for reversal for evidence erroneously admitted: for an appellate court to prosecution to rebut the same, or evid of a character trait of peacefulness of the (b) contact, w/o consent, b/t any part of the defendant’s body or an object and c) Policy: encourage parties to talk freely in negotiations w/o fearing that those
reverse improperly admitted evidence, there must have been: alleged victim offered by the prosecution in a homicide case to rebut evid that the the genitals or anus of another person; statements will be used against them should the settlement fail. We want to encourage
i) A specific objection alleged victim was the first aggressor; (c) contact, w/o consent, b/t the genitals or anus of the defendant and any settlements w/ the belief that they alleviate courts.
(a) Must object correctly and it must be specific as to the grounds for (4) EXCEPTION (3) Character of witness—Evid of the character of a witness, as part of another person’s body; 7) Relevance Exception: Offers to Pay Medical Expenses Occasioned by an Injury
exclusion. provided in Rules 607, 608, and 609. (d) deriving sexual pleasure or gratification from the infliction of death, bodily a) FRE 409: "evid of paying, promising, or offering to pay medical, hospital, or similar
(b) Have to show the evidence could not be excluded for any reason. (a) Note: Look to impeachment section for these rules injury, or physical pain on another person; or expenses occasioned by an injury is not admissible to prove liability for the injury."
(2) Failure to object is considered to be a waiver of any existing ground for (b) Could include, in a civil case, to prove knowledge or lack thereof (e) an attempt or conspiracy to engage in conduct described in paragraphs b) Rationale: want to encourage people to help.
objection, and the evidence is usually admitted. (c) General Explanation: Character evidence is admissible as to a witness's (1)-(4) c) NOTE: Does not exclude collateral admissions of liability (Car accident and person
(3) Waiving right to object Opening the Door – a party may be held to have credibility or lack thereof. Any witness may be impeached by showing her poor vi) SEX CRIMES – Child Molestation: FRE 414 says "It's my fault, and I’ll pay for your expenses" – here, the “I’ll pay” part is inadmissible,
waived the right to object as a consequence of her own tactics in presenting reputation for truth and veracity, or a prior felony conviction. A witness who has (1) General Rule: Basically, in child molestation cases, evidence that defendant but the “it’s all my fault part is admissible.
evidence been impeached may be rehabilitated by showing her good reputation for committed past offenses of child molestation is admissible for any relevant matter, 8) Relevance Exception: Guilty/No Contest Plea
(a) Introducing part of transaction – right to object to cross examination or veracity. doesn’t matter if there was a conviction. a) General Rule FRE 410: In any civil or criminal proceeding, the following is
introduction of rebuttal evidence as to any other part of same transaction (5) Exceptions stated in another way (2) Notice: If the government intends to produce such evidence, it needs to inadmissible against the defendant who made the plea or was a participant in the plea
waived (a) By Criminal accused: disclose that evidence to the defendant 15 days before trial, or at a later time if the discussions:
(b) Introducing inadmissible evidence – P is precluded from objecting to the (i) To show criminal accused’s good character (or to rebut the same) court allows it i) A plea of guilty which was later withdrawn;
admissibility of evidence offered by D to rebut evidence offered by P and (ii) To show victim's character (or to rebut the same) (3) Definition: Rule defines “child” and “offense of child molestation” in subsection (1) Rationale: Would undermine the purpose of the ability to take away the guilty
wrongly admitted (b) By prosecution (d). “ ‘child’ means a person below the age of fourteen…” (Basically, same thing plea. Unfair to bind a Δ to the plea + to use the plea as evid to prove his guilt
ii) Timely made (i) AFTER D’S ATTACK ON VICTIM’S CHARACTER: To show criminal as sexual assault, but to a child): ii) A plea of nolo contendere;
(1) opponent must object timely or the objection is waived. Must object at the first accused's bad character after accused has attacked the victim's character (a) any conduct proscribed by chapter 109A of title 18, USC, that was (1) Rationale: Δ is not actually admitting guilt, so less probative value. Also, may
reasonable opportunity. on same grounds. committed in relation to a child; have cases where Δ wants to accept criminal consequences and doesn’t want to
(a) Can be done before the trial with Motions in Limine: means by which the (ii) HOMICIDE CASE: VICTIM’S PEACEFULNESS: To show character (b) any conduct proscribed by chapter 110 of title 18, USC; be subject to criminal liability.
party can anticipate and ask the court to resolve issues before they arise. trait of peacefulness of alleged victim offered by prosecution in a homicide (c) contact b/t any part of the defendant’s body or an object and the genitals iii) Any statement made in the course of any proceedings under Rule 11 of the Fed.
iii) On a valid ground for objecting and case to rebut evidence that the alleged victim was the first agressor or anus of a child; R. Crim. Proc. or comparable state procedure regarding either of the foregoing pleas;
(1) If you did not point out the right ground will not be reversed on appeal. (d) contact b/t the genitals or anus of the defendant and any part of the body or
(iii) TO REBUT CRIMINAL ACCUSED’S CHARACTER: To rebut Criminal
iv) Error in admitting the evidence that was prejudicial of a child
accused's admissible character evidence iv) Any statement made in the course of plea discussions with any attorney for the
c) Grounds for reversal for evidence erroneously excluded: for an appellate court
(c) By accused in civil case (e) deriving sexual pleasure or gratification from the infliction of death, bodily prosecuting authority which do not result in a plea of guilty or which result in a plea of
to reverse erroneously excluded evidence, there must be: (i) If case involves Fraud or another tort involving moral turpitude, injury, or physical pain on a child; or guilty later withdrawn.
i) No valid ground for objection (f) an attempt or conspiracy to engage in conduct described in paragraphs
defendant can introduce evidence of good character to rebut the charge b) Exception from FRE 410: Such a statements is admissible:
ii) An offer of proof was made, preserving the issue for appeal (Perrin v. Anderson) (1)-(5) i) If another statement from the same discussion has been introduced, and they
(1) Offer of Proof: making a formal record of the substance of the excluded iii) FRE 404(b), Other crimes, wrongs, or acts: “Evid of other crimes, wrongs, or vii) SEX CRIMES – CIVIL CASES of Sexual Assault or Child Molestation: FRE ought in fairness be considered together
evidence. acts is not admissible to prove the character of a person in order to show action in 415 ii) OR, in a criminal proceeding for perjury or false statement if the statement was
(a) How: Lawyer can tell the court what the witness would have said. OR take conformity therewith. (1) Generally Admissible: In a civil case in which a claim is predicated on the made by the defendant under oath, on the record, and in the presence of counsel.
the witnesses testimony outside the presence of the jury. Item marked for (1) EXCEPTION: It may, however, be admissible for other purposes, such as" commission of sexual assault or child molestation, evidence of past acts of sexual 9) Relevance Exception: Insurance
identification and placed in the record. assault or child molestation is admissible
proof of motive, opportunity, intent, preparation, plan, knowledge, identity, or a) FRE 411: "Evid that person was or was not insured against liability is not admissible
(b) Why: Want a record for the appellate court to be able to review on appeal. absence of mistake or accident, provided that upon request by the accused, the (2) Notice: The party intending to offer such evidence needs to disclose the upon the issue whether the person acted negligently or otherwise wrongfully."
(2) Exception: When the substance of the evidence is apparent from the context prosecution in a criminal case shall provide reasonable notice in advance of trial, evidence to the other party 15 days before trial, or at a later time if the court allows i) Other purposes: However, this does not exclude this evidence when offered for
of the questioning, no offer is necessary or during trial if the court excuses pretrial notice on good cause shown, of the it another purpose, such as proof of agency, ownership, control, bias, or prejudice of a
iii) The error in excluding the evidence was prejudicial general nature of any such evidence it intends to introduce at trial.” d) SECOND ISSUE – IS THE PROPER FORM OF CHARACTER EVIDENCE BEING witness.
d) Responding to Objection: If you object, then the opponent must show why it is (2) KIPPOMIA: (note, must be offered for a specific point, can’t get the evidence OFFERED? b) Rationale: Don't want people to be seen as willing to take risks b/c they have
admissible, if it meets an exception to allow admission. in then use it as proof of bad character) i) GENERAL NOTE: Types of Character Evidence that might be Admissible insurance. Also don't want jury to give excessive awards if they believe the insurance
e) Prospects on Appeal: (a) Knowledge (1) Opinion testimony by acquaintances (must personally know person) comp will pay.
i) Appellate courts make it as hard as possible to reverse a trial court’s evidentiary (b) Intent (2) Testimony as to the person's reputation in the community (must personally
ruling. (i) (Crime must require a specific intent, e.g, larceny) know of reputation) ✔ Hearsay
ii) Party complaining on appeal to a judge’s ruling on evidence must show they did (c) Plan (3) Testimony or proof as to specific acts of the person that reflects on the trait
everything they could to bring to inform the judge of the rule and of its application to (i) (admissible to prove existence of larger continuing plan, scheme, or involved 1) Introduction:
the evidence in questions. conspiracy of which the present crime charged is a part) ii) FRE 405(a), Reputation or opinion: “In all cases in which evid of character or a a) A witness's testimony may be based on direct knowledge
✔ Relevancy (d)
(e)
Preparation
Opportunity
trait of character of a person is admissible, proof may be made by testimony as to
reputation or by testimony in the form of an opinion. On cross-examination, inquiry is
i)
(1)
Generally admissible if:
Relevant
(f) Motive allowable into relevant specific instances of conduct.” (2) opinion, conclusions, or estimates
1) Introduction: (g) Identity ii) Sometimes admissible, sometimes not:
iii) FRE 405(b), Specific instances of conduct: “In cases in which character or a
a) Rule 402 – Generally, relevant evidence is admissible, and irrelevant is inadmissible. (i) Admissible if the modus operandi (particular method of crime) in both trait of character of a person is an essential element of a charge, claim, or defense, (1) Hearsay - knowledge based on reports of others
b) Rule 401 – Relevancy under the FRE is defined as “evidence having any tendency crimes are similar and unusual enough to indicate that the same person proof may also be made of specific instances of that person’s conduct.” AKA “ultimate 2) General Rule: If evidence is hearsay, it is prima facie inadmissible. Therefore, we must
to make the existence of any fact that is of consequence to the determination of the action perpetrated both (Criminal signature) fact” determine whether the particular evidence constitutes hearsay, and if so, whether it is
more probable or less probable than it would be without the evidence.”Therefore, there (h) Absence of mistake or accident (1) Defamation case—character evid can be used as an affirmative defense that nonetheless admissible under an exception to the hearsay rule.
must be both materiality and probativeness. (3) Quantum of Proof: Huddleston v. US - Δ only spoke the truth 3) Rationale for excluding hearsay
i) Materiality: to be material, the evidence must be offered to prove a matter in (a) No preliminary finding by the court of whether defendant committed the e) THIRD ISSUE: HABIT EVIDENCE a) Likely to be unreliable, because the person was not under oath, and the four
issue other acts is necessary, only that enough evidence is introduced to permit a testimonial qualities are questionable in hearsay: Perception, Memory, Sincerity,
i) DEFINED: Habit refers to a person's specific, routine and continuous behavior in
ii) Probativeness: to be probative, the evidence must logically tend to prove the reasonable juror to find that the defendant committed the other acts. particular situations Narration/meaning.
proposition for which it is offered. (b) Thus no quantum of proof of the other crimes is required. (1) Factors: 4) Defined: "Hearsay is a statement, other than one made by the declarant while testifying
(1) Note: To be relevant, evidence need not be determinative, it need only make (c) FRE 104(b) - governs the reception of other crimes evidence. Thus, evid (a) Similarity of situation: The more specific the behavior, the more likely it is at the trial or hearing, offered in evidence to prove the truth of the matter asserted"
the fact somewhat more likely; this is because it is the jury that weighs the whose relevancy depends upon the fulfillment of a condition of fact is a) In Elemental Form:
to be deemed a habit
sufficiency of evidence. admissible when there is sufficient evid to support a finding that the condition is (b) Consistency i) A Statement
c) Limitation- Admissible Only to Admissible Purpose - Rule 105: fulfilled. (i) Some courts require that a habit be so routine that it be "invariable." (1) FRE 801(a) - "(1) an oral or written assertion or(2) nonverbal conduct of a
i) Language 105 – “When evidence which is admissible as to one party or for one iv) SEX CRIMES – Past Sexual Behavior/Predispodition (ii) However, other courts, as in the Perrin v. Anderson Court, held that person, if it is intended by the person as an assertion."
purpose but not admissible as to another party or for another purpose is admitted, the (1) Procedure: "habit" stretches to non-routine acts such as an individual's repeated violent (a) ASK – What this intended as a communication?
court, upon request, shall restrict the evidence to its proper scope and instruct the jury (a) Party intending to offer evidence under subsection (b) must file written reaction to encounters with police officers. (b) Note: look below for assertive vs. non-assertive issue
accordingly: motion at least 14 days before trial, specifically describing evidence and (c) Unreflective behavior: The more unreflective or semi-automatic the (c) Silence, if intended to be an assertion
ii) Admissible to one purpose, but inadmissible to another: (pay attention to what purpose, and must serve the motion on all parties and notify the alleged victim ii) Other than one made by the declarant while testifying at the trial or hearing
behavior is, the more likely it is to be a habit
purpose!) (b) Before admitting evidence under this rule, the court must conduct a (1) Declarant 801(b) – “a person who makes a statement”
iii) The trial judge must instruct the jury to consider the evidence only for the
ii) Language FRE 406 - Habit or Routine Practice: "Evid of the habit of a person or
hearing on camera and afford the victim and parties a right to attend and be of the routine practice of an organization, whether corroborated or not and regardless (2) AKA – made or done by someone other than the a testifying witness, OR that
admissible purpose. heard. The motion, related papers, and the record of the hearing must be witness’s prior statement
iv) If the admissible evidence is highly prejudicial to one party but not to the other of the presence of eyewitnesses, is relevant to prove that the conduct of the person of
sealed until the court orders otherwise. organization on a particular occasion was in conformity with the habit or routine iii) That is offered in evidence to prove the truth of the matter asserted
party, the trial court can order a severance. (c) Rationale: safeguard victim against the invasion of privacy, 5) Not Hearsay by Definition:
practice."
2) Relevance Exception: Prejudicial Effect embarrassment and sexual stereotyping. Encourages victims to institute and to a) Statements not offered for truth of the matter asserted
(1) Rationale: habit evid is more probative and a better predictor of what actually
a) Under Rule 403, the trial judge has broad discretion to exclude even relevant participate in legal proceedings against alleged offenders
happened under the circumstances than character evidence. i) Performative Utterance? Significance of speaking - the statement is significant,
evidence when its probative value is substantially outweighed by danger of Unfair (d) Burden: Burden is on the person who wants to admit the evidence to not because of content, but because of the fact that a statement was uttered in the first
Prejudice, Confusion of the issues, Misleading the jury, Considerations of undue delay, show it should be admitted (differs from most evid rules)
iii) Admissible forms of evidence: The FRE is intentionally vague as to the forms
of habit evidence which are admissible; leaving the issue up to the courts. The main place
waste of time, or needless presentation of cumulative evidence. (2) Note: Rape is (1) forceable intercourse, (2) without consent ii) Statements that are a verbal act or operative fact that has independent legal
point, however, is that habit evidence may be offered to show that the conduct of a
i) In Old Chief we learned that the trial court will also look at the need for the (3) INADMISSIBLE: FRE 412(a), Evidence generally inadmissible: “The person on a particular occasion was in conformity with the habit or routine practice. significance
evidence in light of other possible substitutes, and compare the discounted probative following evid is not admissible in any civil or criminal proceeding involving alleged iv) Common Purposes: (1) The statement is a verbal act or an operative fact that gives rise to legal
value of the substitute with the danger of unfair prejudice. sexual misconduct except as provided in subdivisions (b) and (c): consequences
(1) The doing of a particular act in accordance with the habit
b) Language of FRE 403: Exclusion of Relevant Evidence on Grounds of Prejudice, (a) Other Sexual Behavior: (1) Evid offered to prove that any alleged victim (2) To show the standard of care in negligence cases (a) e.g., Contract, defamation, conspiracy
Confusion, or Waste of Time: “Although relevant, evidence may be excluded if its engaged in other sexual behavior; (3) To illuminate the terms of a contract. (2) Also includes verbal parts of the act
probative value is substantially outweighed by the danger of unfair prejudice, confusion of (b) Sexual Predisposition (2) Evid offered to prove any alleged victim’s 4) Relevance Exception: Similar Happenings (a) Words that accompany an ambiguous physical act (the words that
the issues, or misleading the jury, or by considerations of undue delay, waste of time, or sexual predisposition” accompany the payment give the payment its particular legal effect)
iii) Statements that circumstantially or indirectly reveal the declarant's state of mind of subdivision (b). In making its determination it is not bound by the rules of evidence participated. (i) Anticipation of Litigation: The fact that the record is prepared in
(1) E.g., knowledge, fear, emotion except those with regard to privileges. d) Statements for the Purposes of Medical Diagnosis or Treatment anticipation of litigation is not ALONE a reason to find it untrustworthy.
(2) E.g., “my husband is a liar and a thief” – ok because it shows she dislikes ii) Hearsay within Hearsay i) Language FRE 803(4): "The following are not excluded by the hearsay rule, (ii) Medical records: Medical records are relied upon as accurate for their
him, not that he is a thief (1) FRE 805: "Hearsay included within hearsay is not excluded under the even though the declarant is available as a witness …Statements made for purposes own interests, but are only admissible if above elements satisfied, namely
(3) Note on directly reveals - This category doesn’t include direct out of court hearsay rule if each part of the combined statements conforms with an exception of medical diagnosis or treatment and describing medical history, or past or present regular practice of the business and with a real business purpose. U.S. v.
assertions by the declarant as to her state of mind (e.g., "I hate Fred") to the hearsay rule provided in these rules." symptoms, pain, or sensations, or the inception or general character of the cause or Duncan
(a) However, these fit under an exception for declarations of state of mind iii) Confrontation Clause - 6th Amendment external source thereof insofar as reasonably pertinent to diagnosis or treatment" (iii) Computer Records: Computer records are admissible if input
(see later in outline) (1) EVEN if statement fits within a hearsay exception, a criminal defendant can (1) Rationale: Reliable because we assume that people will tell the truth in order procedures conform to standard practice. Hahnemann Univ Hosp v.
iv) Words offered to show their effect on the hearer or reader and not to prove the still object on the grounds that the testimony would violate the confrontation clause. to seek the correct medical treatment Dudnick
truth of the matter asserted (a) Confrontation clause - in criminal cases, the accused has the right "to be (2) Elements: ii) Absence of Records Exception
(1) E.g., notice, knowledge, motive, good faith, duress, probable cause, or that confronted with the witnesses against him." (a) Statement is made for purpose of medical diagnosis or treatment (1) Language FRE 803(7): “The following are not excluded by the hearsay rule,
he had acquired information that had a bearing on his subsequent conduct (b) Ohio v. Roberts two pronged test: In order to admit hearsay without (i) Could be to any medical staff, whether treating or only testifying even though the declarant is available as a witness… Evidence that a matter is not
(2) Words offered to prove motive or anger reflecting on the hearer's later violating the Confrontation Clause: (ii) Doesn't exclude statements made by a third person (friend/relative) included in the memoranda reports, records, or data compilations, in any form, kept
conduct (i) the Declarant must be unavailable, and (b) Statement describes medical history or past or present symptoms, pain, in accordance with the provisions of paragraph (6), to prove the nonoccurrence or
(3) Words to prove that subsequent conduct was in good faith (ii) the hearsay must be reliable (trustworthy) or sensations, or the inception or general character of the cause or external nonexistence of the matter, if the matter was of the kind of which a memorandum,
(4) Warning signs that show knowledge of condition (iii) Idaho v. Wright: consider the totality of the circumstances - those that source thereof report, record, or data compilation was regularly made and preserved, unless the
b) Assertive and nonassertive conduct surround the making of the statement and that render the declarant (c) Statement must be reasonably pertinent to diagnosis or treatment sources of information or other circumstances indicate lack of trustworthiness.”
i) Assertive Conduct: conduct, verbal or nonverbal, which was intended to be an particularly trustworthy, but corroborative evidence may not be considered (i) Therefore, statements about the surrounding circumstances, which (a) Elements:
assertion is within the definition of hearsay. in the trustworthiness inquiry aren't pertinent to diagnosis or treatment are excluded (i) Records must be kept in accordance with 803(6)
(1) Silence, if intended to be an assertion (c) Exception: Inadi case and Bourjaily case added: (ii) E.g., Statement that the car was blue, or the car ran a red light is not (ii) The matter must be of the type regularly made and preserved by the
ii) Nonassertive conduct: conduct, verbal or nonverbal, which was not intended to (i) If the hearsay exception is "firmly rooted," then unavailability and pertinent, while a statement that “I was hit by a car” is pertinent. business
be an assertion is not within the definition of hearsay, and is thus admissible to prove reliability need not be considered e) Recorded Recollections: Past and Present (iii) Sources or other circumstances must not indicate a lack of
the declarant's state of mind as well as to prove the truth of the matter asserted. 1. Note: Catchall provision is not "firmly rooted" i) Notes: PRESENT Recollection Refreshed is better than past recollection trustworthiness
(1) Even though the conduct may be, by process of translation or inference, 7) Hearsay Exceptions where Availability of Declarant is Immaterial recorded, so try it first (b) Expert Testimony: The trend is to allow expert testimony as to the
assertive, it is still considered nonassertive conduct under the definition of hearsay. a) Excited Utterances ii) Present Recollection Refreshed: absence of a record, especially in the medical field.
(2) Examples : command, silence (usually), flight from scene, making a bet i) Language - FRE 803(2) - "The following are not excluded by the hearsay rule, (1) Language FRE 612: "…if a witness uses a writing to refresh memory for the g) Public Records and Reports Exception
(3) Rationale: even though the declarant may be available as a witness… A statement relating to a purpose of testifying; either (1) while testifying, or (2) before testifying, if the court i) Generally: Less restrictive the Business Records Exception (803(6))
(a) No need to worry about the declarant's veracity, since the conduct was startling event or condition made while the declarant was under stress of excitement in its discretion determines it is necessary in the interests of justice, an adverse (1) No “regularly recorded” requirement
not consciously assertive, and thus did not involve veracity. caused by the event or condition" party is entitled to inspect the writing at the hearing, to cross-examine the witness (2) No sponsoring witness needed if certified
(b) There is an assurance of trustworthiness (AKA, actions speak louder than (1) Elements: thereon, and introduce those portions which relate to the testimony of the witness. (3) Use of public records allows the person offering the evidence to put in
words) (a) Exciting event or condition Only those portions that contain matters related to the subject matter of the evidence based on reports of someone NOT in public office
c) Non-human evidence (i) Bootstrapable: Here, this can be bootstrapped; in other words, 104(b) testimony will be allowed, with the remainder preserved in the event of an appeal." ii) Language 803(8): “The following are not excluded by the hearsay rule, even
i) Testimony by a witness as to "statements" made by nonhuman declarants (e.g., can be fulfilled by the statement itself if the surrounding facts and (a) Rationale: because the witness cannot recall, this is the only choice though the declarant is available as a witness… Records, reports, statements, or data
machines, bloodhounds, etc.) does not violate the rule against hearsay circumstances impart a reasonable measure of corroboration (goes to the necessity element). compilations, in any form, of public offices or agencies, setting forth (A) the activities of
d) FRE Exemptions from Hearsay Definition: (b) Which caused declarant to be under stress of excitement (b) Note: This actually isn't evidence, it is merely a stimulus to produce the office or agency, or (B) matters observed pursuant to duty imposed by law as to
i) Intro: FRE 801(d) exempts from the rule against hearsay several categories of (i) Note: Identity of declarant - this is not an element, but must be able to evidence in the form of testimony which matters there was a duty to report, excluding, however, in criminal cases
statements which are not to be considered hearsay at all, even if offered to prove the prove that the declarant (whoever it was) was a witness to the event (c) Elements: matters observed by police officers and other law enforcement personnel, or (C) in civil
truth of their assertions: (c) Statement must have been made while declarant was under stress of the (i) Must lay a foundation: actions and proceedings and against the Government in criminal cases, factual
ii) Prior Statement that is Inconsistent/Consistent/Identification: 801(d)(1)(A- excitement 1. Witness testifies to an inability to recall a fact or event (goes to the findings resulting from an investigation made pursuant to authority granted by law,
C): (i) AKA - Must be "spontaneous" "necessity element") unless the sources of information or other circumstances indicate lack of
(1) General idea: these are treated under the FRE as non-hearsay, and thus as (ii) Note: The more detailed and composed, the less likely it is 2. Witness indicates that a certain writing or object could help refresh trustworthiness."
substantive proof spontaneous/made under stress of excitement his or her memory (1) Rationale:
(2) Rationale: considered to be more reliable than in-court statement b/c in-court (d) Statement must be related to the exciting event or condition 3. Proponent has the writing marked as an exhibit for id and shows (a) Government officials and Reports are generally considered reliable and
statements are tainted by the passage of time ii) Rationale: The availability of the declarant is immaterial. the writing to the witness accurate.
(3) Rule: “A statement is not hearsay if…(1)The declarant testifies at the trial or b) Present Sense Impression 4. Proponent asks the witness to read the writing silently (b) Necessary because public officials handle many records and may forget
hearing and is subject to cross-examination concerning the statement, and the i) Language FRE 803(1) -" The following are not excluded by the hearsay rule, 5. Witness testifies that the document has revived a forgotten facts
statement is: even though the declarant may be available as a witness… A statement describing or memory of a fact or event (2) Notes:
(a) (A) prior inconsistent statements: “inconsistent with the declarant’s explaining an event or condition made while the declarant was perceiving the event or 6. Witness then testifies to his or her recollection of the fact or event (a) 3rd party statements: Watch for statements in such reports, by those who
testimony, and was given under oath subject to the penalty of perjury at a trial, condition, or immediately thereafter…" are not an agent of the government, or whom does not have a duty to report,
(ii) Must furnish the writing to the Opposing side for the opportunity to (1)
which will then creating multiple layers of hearsay requiring another exception
hearing, or other proceeding, or in a deposition (1) Rationale: Availability of Declarant is Immaterial, and is not tainted by cross-examine based on the writing, and (2) introduce part of the writing
(b) (B) prior consistent statements: “consistent with the declarant’s testimony reflection of consequences or by memory loss. (i) Report by one gov’t agent to another
into evid.
and is offered to rebut an express or implied charge against the declarant of (2) Elements: 1. If gov’t employee A tells facts to employee B who writes them up in
iii) Past Recollection Recorded: a gov’t report, A’s statements will be admissible if A had a duty to give
recent fabrication or improper influence or motive (a) Statement
(1) Language FRE 803(5): The following are not excluded by the hearsay rule, the report to B
(c) (C) Prior Identification of a person: “one of identification of a person made (i) FRE 801(a) - "(1) an oral or written assertion or(2) nonverbal conduct
even though the declarant is available as a witness… A memorandum or record
after perceiving the person.” of a person, if it is intended by the person as an assertion." (ii) Statement by one without a duty to talk
concerning a matter about which a witness once had knowledge but now has 1. If the information is supplied by someone who does not work for
(4) Note: Look to Impeachment Rules, if applicable (b) Statement must describe or explain an event or condition insufficient recollection to enable the witness to testify full and accurately, shown to
(c) Immediacy the gov’t and does not have a duty, the report may not include the
iii) Admissions by a part opponent 801(d)(2): have been made or adopted by the witness when the matter was fresh in the statement unless another exception applies
(1) Basic Idea - those statements made by you or your representatives are (i) Statement must be made while the declarant was perceiving the event witness' memory and to reflect that knowledge correctly. If admitted, the
or soon thereafter (3) Elements
useable against you. memorandum or record may be read into evidence but may not itself be received (a) A record, in any form, of public offices or agencies
(ii) Virtually no time may pass between the event being perceived and the
(2) Rationale: Supports the theory of justice—if an opponent says something
declarant's statement about it (This is in contrast to the excited utterance
as an exhibit unless offered by an adverse party" (b) Which sets forth:
against him/her, you should be able to use it.
exception)
(a) Rationale: Writing was at time when witness' recollection was fresh (goes (i) (A)”The activities of the office or agency”
(3) Rule: “A statement is not hearsay if….(2) The statement is offered against a to the reliability element). Additionally, because the witness lacks sufficient 1. Note: Must be offered to show that those activities occurred
(3) Bootstrapping is acceptable – look at FRE 104(a), here, the court can base
memory to testify, the only choice is to admit the hearsay written statement
party and is 2. Note: No requirement of being based on personal knowledge of
its admissibility determination on the hearsay statement itself) (goes to the necessity element).
(a) Own statement: “(A) the party’s own statement, in either an individual or a
ii) "Professor Waltz's General Rules:" (These just give good explanations of the the reporting officer
representative capacity” (b) Elements: 3. Ex: Records of receipts, payroll docs, personnel records,
above rule) (i) Witness must have personal knowledge of the fact or event recorded
(b) Adoptive admission “(B) a statement of which the party has manifested an (1) Declaration need not cause excitement disbursements of gov. monies
adoption or belief in its truth’ (2) It need not relate to the principal litigated event
(ii) Witness has insufficient recollection to testify fully and accurately (ii) OR (B) “Matters observed pursuant to duty imposed by law as to
(i) Silence as admission - In US v. Hoosier, the court stated that silence about the matter which matters there was a duty to report”
(3) Speaker need not be a participant in the perceived event
can be an implied admission. This requires: (4) Speaker must have been a parcipient witness (iii) Witness must have prepared or adopted the record at a time when the 1. Observations admissible if made in the line of duty and the official
1. The party must have been present, and capable of hearing and (5) Speaker need not be id’d fact or event was fresh in her memory had a duty to report those observations
understanding the accusations (6) Speaker need not be oath-worthy (iv) Record must be an accurate recording of the witness' perception of 2. Criminal Cases Exception: “excluding, however, in criminal cases
2. The party must have been physically and mentally able to deny the (7) Subject matter is limited to the observed event the fact or event matters observed by police officers and other law enforcement
accusations (8) Minimal time lapse is permissible (v) Document must be the authentic memorandum which has not been personnel.
3. A reasonable person would have denied the accusatory statement (9) Statement still admissible if the speaker is available tampered with a. "Other law enforcement personnel" includes any officer or
under the particular circumstances (10) Impressions in the form of an opinion are also available f) Business Records Exception employee of a governmental agency which has law enforcement
4. Note: not applicable after miranda rights have been read, because (11) Doesn’t matter if the speaker is on the witness stand i) Language FRE 803(6): “The following are not excluded by the hearsay rule, responsibilities
person has right to be silent. c) Then Existing Mental/Emotional/Physical Condition even though the declarant is available as a witness… A memorandum, report, record, i. Ex: I.N.S. agents, border inspectors, but not building
(c) Authorized admission: “(C) a statement by a person authorized by the i) Language FRE 803(3): "The following are not excluded by the hearsay rule, or data compilation, in any form, of acts, events, conditions, opinions, or diagnoses, inspectors, medical examiners, judges
party to make a statement concerning the subject” even though the declarant is available as a witness … A statement of the declarant's made at or near the time by, or from info transmitted by, a person w/ knowledge, if b. Evaluative Reports: It is the "clear intent of congress to make
(i) non bootstrappable – have to prove authority separately - see below then existing state of mind, emotion, sensation, or physical condition (such as intent, kept in the course of a regularly conducted business activity, and if it was the regular evaluative law enforcement reports absolutely inadmissible in
(ii) Authorization can be express or implied plan, motive, design, mental feeling, pain, and bodily health), but not including a practice of that business activity to make the memorandum, report, record or data criminal cases." - U.S. v. Oates
(d) Admissions by Ee/agent: “(C) a statement by a person authorized by the statement of memory or belief to prove the fact remembered or believed unless it compilation, all as shown by the testimony of the custodian or other qualified witness, c. Exception: Routine Nonadversarial Reports: Several circuits
party to make a statement concerning the subject” relates to the execution, revocation, identification, or terms of declarant's will." or by certification that compiles with Rule 902(11), 902(12), or a statute permitting allow the admission of reports of the observations of law
(i) non bootstrappable – have to separately prove agency or employment (1) Rationale: Such declarations have some degree of spontaneity certification, unless the source of info or the method or circumstances of preparation enforcement officers against criminal defendants when the reports
relationship and scope thereof - see below (trustworthiness), and may be the best evidence of the declarant's state of mind indicate lack of trustworthiness. The term, “business” as used in this paragraph are of "routine, nonadversarial matters."
(ii) Must be within scope of employment, during employment/agency- (necessity), eliminates need to check the declarant's memory. includes business, institution, association, profession, occupation, and calling of every (iii) OR (C): Findings of an investigation pursuant to legal authority:
principle relationship (2) Elements: kind, whether or not conducted for profit.” 1. Language: “in civil actions and proceedings and against the
(e) Admissions by co-conspirator during the court of and in furtherance of (a) The declarant must have made a statement concerning her then existing (1) Rationale: Align the Rules of evidence with modern business practice, which Government in criminal cases, factual findings resulting from an
conspiracy state of mind, emotion, sensation, or physical condition are large and complex, making it impossible to call everyone who had taken part in investigation made pursuant to authority granted by law, unless the
(i) Non boostrappable – have to separately prove existence of conspiracy (b) However, memory or belief used to prove the occurrence of a past event the making of the business record (goes to the necessity element). sources of information or other circumstances indicate lack of
and the participation therein of the declarand and the party against who the unless it relates to a will (2) Notes: trustworthiness."
statement is offered - see below (i) NOT: Proof of Occurrence: Statement cannot be used to prove (a) Can include acts, events, conditions, opinions, or diagnoses 2. Notes:
(f) DEFINITION: ‘Non-bootstrappable” occurrence, because then you are using the statement as a belief to prove (b) Can be in any form a. Admissible whether facts or opinions
the fact believed (3) Elements: b. Civil Cases: Admissible
(i) Basically, independent evidence must support the finding of these
(a) Records in question were maintained by a "business"
conditional circumstances. (ii) NOT: Intent of Third Party: Cannot be used to prove the intent of a c. Criminal Cases: Admissible only against the government
(i) Broad Definition: "The term, “business” as used in this paragraph 3. Trustworthiness Exception: the FRE provides that the findings
(ii) Rule Language 802 under 1997 ammendment - The contents of the third party, because then you are using the statement as a belief to prove
includes business, institution, association, profession, occupation, and should be excluded if the opponent demonstrates that the "sources of
statement shall be considered but are not alone sufficient to establish the the fact believed
calling of every kind, whether or not conducted for profit.” information or other circumstances indicate lack of trustworthiness"
condition precedent’s under [801(d)(2)(C,D,E)], being the declarant’s (c) Prove a Subsequent Act (Hillmon Doctrine)
(b) It was the regular practice of the business to keep this type of record
authority under subdivision (C), the agency or emmployment relationship (i) The Hilmon court held that a person's out of court declarations of state a. Factors court may consider:
(i) Doesn't mean it has to be frequently, just regular routine i. The timeliness of the investigation
and scope thereof under subdivision (D), or the existence of the conspiracy of mind are admissible to show the probability that the person committed (ii) Accident reports are not systematic conduct of a business.
and the participation therein of the declarant and the party against whom some subsequent act pursuant to that declared state of mind. ii. The special skill or experience of the official
(iii) Rationale: Goes to the trustworthiness element iii. Whether a hearing was held and the level at which
the statement is offered under subdivision (E). (3) Memory or Belief of Past Occurrence/Wills (c) The entries in the record were made close to the time of the event to
(4) Note: This is not to be confused w/ a declaration against interest (a) Statements of memory or belief to prove the fact remembered or believed conducted, and
which they relate iv. Possible motivation problems
(5) Note: This rule is not concerned with the following: are excluded from this exception to hearsay, unless it involves the execution, (d) The source of the information must have had personal knowledge
(a) Doesn’t matter whether the statement was based on personal knowledge revocation, identification, or terms of the declarant's will. iii) Records of Vital Statistics
(i) 3rd Party: Statements by one not part of a business are not admissible (1) Language FRE 803(9): "The following are not excluded by the hearsay rule,
(b) Doesn’t matter if statement is a normally inadmissible lay opinion (4) When statement includes SURROUNDING CIRCUMSTANCES: for the truth for the matter asserted. U.S. v. Vigneau
(c) Doesn’t matter if the party opponent is actually available to testify (a) Surrounding Circumstances Included: If a statement of present mental even though the declarant is available as a witness… Records or data
(e) There must be a real business purpose for accurately recording the compilations, in any form, of births, fetal deaths, deaths, or marriages, if the report
6) HEARSAY EXCEPTIONS state includes reference to surrounding circumstances, the entire statement will particular piece of info the party wants to rely on thereof was made to a public office pursuant to requirements of law.
a) This is Hearsay but excepted from the general prohibition of the evidence normally be admitted, but with a limiting instruction (f) There is a proper authenticating witness on the stand who knows about
(b) MAYBE: Act of Third Party: If the statement of present intent concerns an (2) Rationale: Such events are required by law to be reported as they happen,
b) General Definitions/Concerns: this business record and can supply the other elements of the foundation
act which requires the cooperation of another, most courts will allow the thus assuring their trustworthiness.
i) Questions of Admissibility Generally FRE 104(a) "Preliminary questions (i) 'Custodian of the Records' would be most appropriate iv) Absence of Public Record or Entry:
concerning the qualification of a person to be a witness, the existence of a privilege, or statements, but will usually require that there be independent evidence either (g) The record is trustworthy considering the source, method or
the admissibility of evidence shall be determined by the court, subject to the provisions that the declarant actually did the act, or that the third person actually (1) Language FRE 803(10): "The following are not excluded by the hearsay rule,
circumstances of preparation even though the declarant is available as a witness… To prove the absence of a
record, reports, statements, or data compilation, in any form, or the nonoccurrence (i) Opportunity - must have been present with chance to develop the testimony as evidence of the matter it is offered to prove? i) HOWEVER: Party may not put a witness on a stand for the sole purpose of
or nonexistence of a matter of which a record, report, statement, or data testimony (6) Opposing party must be provided with notice of the proponent's intended use impeaching him (U.S. v. Hogan)
compilation, in any form, was regularly made and preserved by a public office or 1. Predecessor in interest - Broad interpretation of the statement before trial. (1) Rationale: jury will hear the impeachment evidence and can be used as a way
agency, evidence in the form of a certification in accordance with Rule 902, or
testimony, that diligent search failed to disclose the record, report, statement, or
(ii) Similar motive - there must be a similar enough overlap between the
issues existing at the time of the prior hearing, and the issues existing at the
✔ Direct/Cross/Impeachment b)
to avoid the hearsay rule
Effect of impeachment
data compilation, or entry." present trial, that the above opportunity for examination was a meaningful i) The fact that a witness has been impeached does not mean that her testimony
(a) For Exam: substitute for cross examination at the present trial 1) Witness Competency: will be stricken or disregarded.
(i) A certificate, or testimony, from the custodian of the public record that c) Dying Declaration a) Assumed Competent: FRE 601 – Every person is competent to be a witness except ii) The weight of the evidence is for the jury to decide.
diligent searched failed to disclose certain records, or testimony as to the i) Language - FRE 804(b)(2) - In a prosecution for homicide or in a civil action or as otherwise provided in these rules. c) No Impeachment on "Collateral Matters" - more narrow than relevancy
same is admissible to prove that the matter has not been reported or proceeding, a statement made by a declarant while believing that the declarant's death i) Exception: State Rules: i) Rule: Impeachment evidence must always be relevant to the witness's credibility
recorded, if it is the kind of matter as to which reports are regularly made by was imminent, concerning the cause or circumstances of what the declarant believed (1) states may have different rules, therefore, if a federal court is applying state or some other issue in the present litigation.
the agency involved. to be impending death is excepted from the general prohibition of hearsay evidence. law, the federal court may have to apply that states different competency (1) Therefore, merely proving that the witness was incorrect about an
h) Judgment of Previous Conviction (1) Elements: requirements unimportant detail is not considered sufficiently relevant to credibility.
i) Language 803(22): "The following are not excluded by the hearsay rule, even (a) If Criminal case – only allowed if homicide ii) Rationale: We want all the relevant evidence in so that the jury can weigh it. (2) Rationale: Probative value outweighed by risks of confusion of issues and
though the declarant is available as a witness… Evidence of a final judgment, entered (b) Declarant must believe death is imminent (doesn’t actually have to be iii) Exception: Judge/Jury status undue consumption of time.
after a trial or upon a plea of guilty (but not upon a plea of nolo contendere), adjudging dead) (1) FRE 605 Competency of Judge: a judge is not competent to testify in same (3) Note: There is no express collateral issue rule in the FRE – Just the
a person guilty of a crime punishable by death or imprisonment in excess of one year, (c) Declarant must be "unavailable" (see definition above) (doesn’t actually case as sitting. Need no objection to preserve for appeal. Relevance Rules and 403 balancing test
to prove any fact essential to sustain the judgment, but not including, when offered by have to be dead) (2) FRE 606 Competency of Jury: d) Methods of Impeachment
the Government in a criminal prosecution for purposes other than impeachment, (d) Statement must relate to the cause/circumstances of impending death (a) When there’s an inquiry into the validity of a verdict or indictment, a juror i) Contradictory Evidence
judgments against persons other than the accused. The pendency of an appeal may (2) Rationale: Declarant is unavailable can’t testify about anything relating to deliberations, but she can testify about: (1) All or part of a witness's testimony may be rebutted by proof of contradictory
be shown but does not affect admissibility." (i) Extraneous prejudicial information improperly brought to bear on any facts, but may not introduce "collateral matters."
(1) Elements:
ii) ADDITIONAL ELEMENT: Personal Knowledge Issue FRE 602:
juror (2) Required Purpose: Must be for the purpose of impeachment; cannot call a
(1) If the declarant did not have adequate opportunity to observe the facts
(a) A person must have been convicted of a crime (ii) Any outside influence improperly brought to bear on any juror witness to the stand primarily for the purpose to get before the jury what would
recounted, declaration will be rejected for lack of firsthand knowledge
(b) Conviction must be final, either entered after a trial or a guilty plea (not (iii) Whether there was a mistake in entering the verdict onto the verdict other wise be inadmissible evidence.
(a) E.g., if person was shot in back, may have no person knowledge of who
including no contest pleas) form
(c) Conviction must be for a felony
shot him ii) Lack of knowledge or perception
(b) Must object to preserve for appeal.
(d) Must be a felony b/c many people cop pleas to misdemeanors out of (2) In other words: Expressions of suspicion or conjecture are to be excluded
(c) Policies – wanting to maintain fairness, and ensuring that the jury’s
(1) A witness's credibility may be attacked by showing that she had no real
when there is merely room for doubt as to whether the statement is based on knowledge of the facts related in the testimony, or that the witness's faculties were
convenience. decision is insulated and inviolate
knowledge, the question is for the jury so dulled at the time of the occurrence that her perception was questionable:
(e) Purpose must be to prove "any fact essential to sustain the judgment" b) Must have Personal Knowledge: FRE 602 : A witness may not testify to a matter
d) Statement Against Interest (a) Asleep, Drunk, Drugs/medication, Poor
(f) Criminal Cases Exception: If used by the Government in a criminal unless evidence is introduced sufficient to support a finding that the witness has personal eyesight/hearing/colorblind/concussion, Preoccupied/distraction, Psychiatric
prosecution for purposes other than impeachment, judgments against persons i) Language FRE 804(b)(3) - "A statement which was at the time of its making so knowledge of the matter.
far contrary to the declarant's pecuniary or proprietary interest, or so far tended to Condition, Poor Memory
other than the accused. i) Proving Personal Knowledge: “Evidence to prove personal knowledge may, but (b) Can be proven by cross examination or by introducing extrinsic evidence
(2) Evidenciary Effect: Two Views subject the declarant to civil or criminal liability, or to render invalid a claim by the need not, consist of the witness’ own testimony. “
(a) Majority - conviction is merely evidence, not conclusive to the facts declarant against another, that a reasonable person in the declarant's position would iii) RELIGION - FRE 610 - "Evidence of the beliefs or opinions of a witness on
(1) AKA - This rule is a specialized application of conditional relevancy, per [Rule matters of religion is not admissible for the purpose of showing that by reason of their
(i) Rationale: the criminal conviction is not res judicata because the not have made the statement unless believing it to be true. A statement tending to 104] “bootstrappable”
expose the declarant to criminal liability and offered to exculpate the accused is not nature the witness' credibility is impared or enhanced."
requisite identity of parties does not exist ii) Exception for Experts: This rule is subject to the provisions of rule 703, relating to
(b) Minority - Criminal conviction is entitled to the res judicata effect of admissible unless corroborating circumstances clearly indicate the trustworthiness of iv) Evidence attacking credibility: Character Impeachment
opinion testimony by expert witnesses.
collateral estoppel, and therefore the criminal conviction is conclusive against D the statement." (1) Witnesses who take the stand put their character for honesty and veracity in
iii) NOTE: Post-hypnotic testimony issue, therefore they can be impeached by evidence that their character is such
as to the facts upon which it is based (1) Rationale: Sincerity factor strong if the statement is against your interest
(2) Elements: (1) Must have to testify that beforehand, they had personal knowledge, and that that they may lie under oath. Evid admissible for this purpose:
ii) Rationale: Because is the facts essential to a judgment are proved beyond a hypnosis is refreshing it.
reasonable doubt, reliability factor is not an issue. (a) The declarant is not a party to the case (a) Poor Reputation or Opinion of Truthfulness
(b) The declarant must be unavailable (see 804(a)) (2) If witness’s recall has been refreshed through hypnosis, Three approaches:
i) Learned Treatises (i) Language FRE 608(a): The credibility of a witness may be attacked or
(a) Does not raise a question of competency, but only a question of credibility
i) Language FRE 803(18): "The following are not excluded by the hearsay rule, (c) The statement must have been sufficiently against important interests of supported by evid in the form of opinion or reputation, but subject to these
for the jury
even though the declarant is available as a witness…To the extent called to the the declarant when made so that a reasonable person in the same position limitations:
(b) Other jurisdictions require that safeguards be complied with before the
attention of an expert witness upon cross-examination or relied upon by the expert would not have made the statement unless she believed it to be true. 1. (1) the evidence may refer only to character for truthfulness or
testimony is admissible
witness in direct examination, statements contained in published treatises, periodicals, (i) "Against interest":
(c) Other’s say it is per se inadmissible
untruthfullness, and
or pamphlets on a subject of history, medicine, or other science or art, established as 1. According to the present weight of state case authority, a 2. (2) evidence of truthful character is admissible only after the
(i) Facts remembered before the hypnosis are OK as long as they were
a reliable authority by the testimony or admission of the witness or by other expert statement against interest only falls within the exception if it is against character of the witness for truthfulness has been attacked by opinion or
documented
testimony or by judicial notice. If admitted, the statements may be read into evidence the declarant's pecuniary (financial) or proprietary (property) interest. reputation evid or otherwise
(d) Criminal D’s right to testify:
but may not be received as exhibits." a. FRE specifically lists a statement that would tend to subject the (b) Specific Instances of noncriminal misconduct Showing Poor
declarant to civil liability to another person (i) It is unconstitutional to have a per se rule of inadmissibility when the
Truthfulness
(1) Elements: witness is the D in the case, because the d has the right to testify in his own
(a) Published work must have been relied on by an expert or called to b. FRE specifically lists a statement that would "render invalid a (i) Language FRE 608(b):
claim by him against another" defense. (only applies do D) (ii) Can't be proved by extrinsic evidence.
attention of an expert witness (ii) Does not mean that post hypnotic recall to be admissible, there just
(b) Statements must come from published treatise, periodical, pamphlet, etc. 2. Additionally, the weight of case authority indicates that the (iii) But, may be inquired into on cross-examination:
cannot be a per se rule of inadmissibility 1. Upon the discretion of the court
(c) The published work must be established as reliable authority, either by prejudice to such interest must be immediate and substantial, and the
declarant must then know, or be chargeable with knowledge that it is iv) NOTE: Remember present recollection refreshed hearsay exception 2. If (1) probative of witness' character truthfulness or untruthfulness,
expert testimony or by judicial notice
(2) If admitted, the statements from the published work may only be read into (rather than merely an opinion or estimate as to the facts). v) NOTE: Remember Past recollection recorded hearsay exception or (2) concerning the character for truthfulness or untruthfulness of
evid a. How disservable does it have to be: Doesn't have to be a full c) 603 – Oath, Understand Duty of Truth: Before testifying, every witness shall be another witness as to which the character of the witness being cross-
j) Statements in Ancient Documents admission of liability, enough that it provides an important link in a required to declare that the witness will testify truthfully, by oath or affirmation examined has testified.
chain of evidence that subjects the declarant to liability. administered in a form calculated to awaken the witness’ conscience and impress the (iv) Giving this testimony does not waive the witness' privilege against self-
i) Language FRE 803(16): "Statements in ancient documents. Statements in a
b. Ridicule/hatred: A statement subjecting the declarant to witness’ mind with the duty to do so. incrimination when examined with regard to matters that relate only to
document in existence twenty years or more, the authenticity of which is established."
ridicule, hatred, or disgrace is not admissible under (b)(3) exception i) NOTE: Child’s age may be determinative of this element character of truthfulness.
(1) Note: Look to ancient documents doctrine, which basically states that if ii) Method:
ancient, it is authentic c. Must be offered in its disserving aspect: Williams v. US - (c) Conviction of a Crime
(1) There is no special verbal formula required, simply a solemn undertaking to (i) 609(a)(1): Felony convictions that do not involve dishonesty or
k) Other Hearsay Exceptions: including (11) Records of Religious Organizations, (12) statements may be self-serving in one aspect and disserving in
tell the truth false statements
Marriage, baptismal, and similar certificates, (13) Family records, (14) Records of another. If so, it must appear that the declaration is being offered in
(2) Judges and clerks may administer the oaths/affirmations 1. Other than Accused: Must be offered against a witness other than
documents affecting an interest in property, (15) Statements in documents affecting an evidence with respect to its disserving aspect.
iii) Rationale: want the witness to convey truthful information the accused in the present case
interest in property, (17) Market reports, commercial publications, (19 Reputation (3) Statement exculpating the accused:
2) Direct examination: 2. Bad Felony: Prior conviction was for a felony punishable by death
concerning personal or family history, (20) Reputation concerning boundaries or general (a) A statement tending to expose the declarant to criminal liability and
a) Point of Direct: or more than one year imprisonment
history, (21) Reputation as to character, (23) Judgment as to personal, family, or general offered to exculpate the accused is not admissible unless corroborating
i) 1st examination of any witness a. Note: A mere showing of arrest or indictment is insufficient
history, or boundaries. circumstances clearly indicate the trustworthiness of the statement.
ii) Intended to establish the facts the proponent needs to establish
8) Hearsay Exceptions Where the Declarant is Unavailable (b) The corroborating circumstances cannot come from the statement alone 3. Time Limit: No longer than 10 years elapsed since date of
b) Objectionable questions: Argumentative, assumes questions not in evidence,
a) Definition of “Unavailable” - FRE 804(a) Unavailability as a witness' includes e) Statement of Personal or Family History conviction OR release of the witness from confinement imposed for that
compound, called for speculation, repetitive, narrative, asked and answered, leading conviction
situations in which the declarant: i) Language FRE 804(b)(4) :
(most notorious) a. OR if the judge determines that the 10-year rule does not
i) Exempted - Is exempted by ruling of the court on the ground of privilege from (1) By Self: (A) A statement concerning the declarant’s own birth, adoption,
c) Narrative Questions Objectionable: apply, the proponent has given the opposing side advance notice of
testifying concerning the subject matter of the declarant's statement marriage, divorce, legitimacy, relationship by blood, adoption, or marriage,
i) Asks witness a question so broad as to invite a narrative its intent to use such evidence.
ii) Refusal - Persists in refusing to testify concerning the subject matter of the ancestry, or other similar fact of personal or family history, even though declarant
had no means of acquiring personal knowledge of the matter stated ii) Exception: Experts generally are allowed to testify in a narrative form 4. No Pardon/Nullification: Conviction has not been nullified (i.e.
declarant's statement despite an order of the court to do so d) Leading Questions Objectionable:
(2) By Related: OR (B) A statement concerning the foregoing matters, and death pardon) or subject to a procedure that nullifies the conviction based on a
iii) Lack of memory - Testifies to a lack of memory of the subject matter of the i) Language FRE 611(c), Leading Questions: “Leading questions should not be finding of rehabilitation where that person has not been convicted of a
declarant's statement also, of another person, if the declarant was related to the other by blood,
adoption, or marriage or was so intimately associated with the other’s family as to used on the direct examination of a witness except as may be necessary to develop subsequent felony
iv) Death or physical/mental Inability - Is unable to be present or to testify at the the witness’ testimony. Ordinarily leading questions should be permitted on cross- 5. Not Prejudicial: Probative value of admitting this evidence
be likely to have accurate information concerning the matter declared.
hearing because of death or then existing physical or mental illness or infirmity examination. When a party calls a hostile witness, an adverse party, or a witness outweighs its prejudicial effect:
(1) Factors for Judge to Consider under FRE 104(a) f) Forfeiture by Wrongdoing:
identified w/ an adverse party, interrogation may be by leading questions.” a. Witness other than criminal accused is governed by 403
i) Note: See definition of unavailability. Here, the person would not be “unavailable”
(a) The declarant's direct statements regarding his feelings or injury ii) Definition Leading Question: suggests the answer in the question (requiring that the probative value is substantially outweighed by the
(b) The nature of the declarant's injury or wound ii) Language FRE 804(b)(6) - A statement offered against a party that has
(1) If an ordinary person gets the impression that the question suggests the prejudicial effect), not 609(a)(1) (requiring any level of prejudicial
(c) Administration of last rights engaged or acquiesced in wrongdoing that was intended to, and did, procure the
answer the questioner wants—it is leading effect outweighing the probative value), thus offering the criminal
(d) The making of a will unavailability of the declarant as a witness.
(1) E.g., person kills a person who is known to be testifying against his mob boss iii) Exceptions—i.e. leading questions are ok when: accused an extra measure of protection.
(e) Statements by 3rd persons such as doctors or nurses that one's death is (1) they concern preliminary matters on issues not in dispute (i.e. for efficiency, b. Factors:
certain or near in a couple weeks
9) Residual Exception speed up trial) i. Whether the prior crime weighs on truthfulness
v) Absence - Is absent from the hearing and the proponent of a statement has been (2) they are used to refresh a witness’ recollection ii. Remoteness of the crime
unable to procure the declarant's attendance (attendance or testimony if under b-2,3, a) General Idea: A statement not specifically covered by Rule 803 or 804 is not
excluded by the hearsay rule if certain factors are met (3) they are directed at a “handicapped” witness—i.e. very young, extremely old, iii. How central credibility is to the case.
or 4) by process or other reasonable means." (ii) 609(a)(2): Convictions involving dishonesty or false statements
b) Language FRE 807 - "A statement not specifically covered by Rule 803 or 804 but or infirm
(1) So you cant' just say that the person is outside the subpoena power of the
court, you must try to get the testimony by deposition or similar means. having equivalent circumstantial guarantees of trustworthiness, is not excluded by the (4) they are directed at a hostile or adverse witness 1. Type of conviction: Prior conviction required proof or admission of
hearsay rule, if the court determines that (A) the statement is offered as evidence of a (5) they are allowed by the trial judge w/in his/her discretion as a means of better an act of dishonesty or false statement, whether a misdemeanor or
vi) Exception for proponent's fault in causing unavailability - a declarant is not
felony
unavailable as a witness if exemption, refusal, claim of lack of memory, inability, or material fact; (B) the statement is more probative on the point for which it is offered than ascertaining the truth, avoiding a waste of time, or protecting a witness from
any other evidence which the proponent can procure through reasonable efforts; and (C) harassment a. Bank robbery and other stealing is not necessarily a crime of
absence is due to the procurement or wrongdoing of the proponent of a statement for dishonesty
the purpose of preventing the witness from attending or testifying. See 804(b)(6) the general purposes of these rules and the interests of justice will best be served by 3) Cross Examination:
admission of the statement into evidence. However, a statement may not be admitted a) Point: second examination of witness b. Test: Whether either (1) dishonesty or false statement is an
b) Former Testimony
element to the crime or (2) dishonesty or false statement was used
i) Language FRE 804(b)(1) - "Testimony given as a witness at another hearing of under this exception unless the proponent of it makes known to the adverse party b) Leading questions: Permissible
sufficiently in advance of trial or hearing to provide the adverse party with fair opportunity c) Scope of Cross-examination: in committing the crime (U.S. v. Brackeen)
the same or a different proceeding, or in a deposition taken in compliance with law in c. Dishonesty - refers only to deceitful behavior
to prepare to meet it, the proponent’s intention to offer the statement and the particulars of i) Language FRE 611(b) - "Cross-examination should be limited to the subject
the course of the same or another proceeding, if the party against whom the testimony 2. Time Limit: No more than 10 years since the date of the conviction
it, including the name and address of the declarant." matter of the direct examination and matters affecting the credibility of the witness.
is now offered, or, in a civil action or proceeding, a predecessor in interest, had an or the release of the witness from the confinement for that conviction.
i) Elements: The court may, in the exercise of discretion, permit inquiry into additional matters as if
opportunity and similar motive to develop the testimony by direct, cross, or redirect OR judge can determine that the 10 year rule does NOT apply; then
examination." (1) It must not be covered by rule 803 or 804 on direct examination.”
(2) There must be circumstantial guarantees of trustworthiness equivalent to proponent has to give the opposing side advance notice of its intent to
(1) Rationale: out of necessity due to the witness being unavailable, and the ii) General rule: Cross is limited to the scope of direct
those of the other hearsay exceptions use such evidence.
testimony is reliable because it was under oath, etc., and the opposing party, or (1) Exception: a large minority of states allow "wide open cross"
(a) Examples: Perception, Memory, Sincerity, Narration/meaning. 3. No Nullification or rehabilitation: The conviction has not been
predecessor in interest, had a chance at the previous trial to develop that (2) Exceptions: Can question the witness about anything related to credibility of
subject to a procedure that nullifies the conviction based on a finding of
testimony and check the witnesses recollection, memory, sincerity and narration. (3) It must be offered as evid of a material fact the witness
(4) It must be more probative on point for which it is offered than any other innocence and has not been the subject of a procedure that nullifies the
(2) Elements: (a) can include bias, motive, interest, untruthfulness, material prior conviction based on a finding of rehabilitation where that person has not
(a) The declarant must be unavailable (see 804(a)) evidence which the proponent can procure through reasonable efforts. inconsistency of a witness been convicted of a subsequent felony.
(b) Testimony must have been given as a witness or in a deposition taken in (5) General purposes of these rules and the interests of justice will best be (3) Judge's Discretion: Judge has discretion to go outside the scope of the direct
compliance with law in the same or a different proceeding served by admission of the statement into evidence.
v) Evidence attacking credibility: Hostility, bias and interest evidence
4) Impeachment (1) Rule: While impeachment by bias is not specifically recognized by the FRE,
(c) The party against whom the testimony is presently offered (or in a civil (a) Examples: a) FRE 607 - "The credibility of a witness may be attacked by any party, including the
(i) Necessity (Unavailability of declarant) the SC has held that it is a permissible method of impeachment under 402
case, a predecessor in interest ), had an opportunity and (2) similar motive, to party calling the witness"
(ii) Is there a reasonably accurate value that can be placed upon the because it is logically relevant to a fact of consequence and is admissible unless
develop the testimony by direct, cross, or redirect examination.
there is a basis for exclusion. admits it as substantive evidence. Under the FRE a prior identification (1) The privilege is asserted against a communication between the spouses originals.
(2) How to prove: May be proved by cross examination or introducing extrinsic is nonhearsay. (a) "observations" are not privileged, because not a "communication" ii) And Is being offered "to prove the contents of the writing"
(2) The communications are confidential
evidence (Rule 611(a)) gives a judge discretion to allow extrinsic evidence to (b) Hearsay status: FRE 801(d)(1)(B) provides that an inconsistent statement (1) Examples of when used to “prove the contents”:
(a) "Confidential" means that the communication must be made outside the presence
prove bias is not hearsay if it is offered to rebut a charge of "recent fabrication or improper of third parties, and it must concern a matter that the communicating spouse would (a) The legal rights, obligations, consequences arise directly from the writing.
(3) Requirement of Foundation: influence or motive." probably desire to be kept secret (b) Contents of writings contain independent significance
(a) Most courts require that before a witness can be impeached by extrinsic
evidence of bias, the cross examiner must first ask about the facts that indicate ✔ Privileges (b) Communications between spouses are presumed privileged, burden on party
objecting to the claim of privilege to show unprivileged. iii)
(c) OR Witness testifying about what the writing says.
Exception ORIGINAL NOT REQUIRED WHEN: FRE 1004
such bias, hostility or adverse interest. Then if the witness denies bias, (3) The communication was made during a valid marriage
(1) Originals are lost or destroyed:
extrinsic evidence may be permitted in the judge's discretion under Rule 1) An important Foreword from Selig Himself: In the 500 series rules, it says that in ii) Important Distinctions: Applies whether or not the couple remains validly married,
(a) All originals are lost or have been destroyed. Unless proponent lost or
611(a). federal courts, you use federal common law. There are differences among state that whether the proceeding is a criminal or civil case
destroyed them in bad faith
vi) Evidence attacking credibility: Prior inconsistent acts or statements recognize some cp privilege. If there is a specific case that we read, then look to that. iii) Rationale: encourage trust and confidence between spouses.
However, on all of these privileges, we only need to know a specific rule if we read a case on (2) Original not obtainable: no original can be obtained by any available judicial
(1) Procedure: Language FRE 613(a): "In examining a witness concerning a iv) Exceptions: Crime against spouse or child, or the furtherance of crime or fraud
process or procedure
prior statement made by the witness, whether written or not, the statement need it. Otherwise, we just need to know that the category exists, and make a rational argument. If v) Waiver: if a specific objection is not made by one entitled to assert the privilege, it is
you see a privileges, then just make an argument as to what you think it should be (so (3) Original in Possession of Opponent:
not be shown nor its contents disclosed to the witness at that time, but on request waived.
(a) At a time when an original was under the control of the party against
the same shall be shown or disclosed to opposing counsel.” elements don’t necessarily matter 100%, nor do any rules) 6) Clergy-Penitent Privilege
2) In general whom offeed, that party was put on notice, by the pleadings or otherwise, that
(a) Basic Rule: when examining a witness concerning a prior statement, the a) Rule: the Clergy-Penitent privilege, which allows either the penitent or the clergy member to the contents would be a subject of proof at the hearing, and that party does not
statement need not be shown or disclosed to the witness, but on request, it a) Who can Claim: Privileges are personal in nature and can only be claimed by the 'holder of the refuse to disclose, or prevent the other from disclosing communications, is established if:
privilege,' which is the person whose interest or relationship is sought to be protected (e.g. client or produce the original at the hearing
shall be shown or disclosed to the opposing counsel. i) There is a communication
patient, sometimes attorney) (1) "observations" are not privileged, because not a "communication" (4) Contents of writing only "collateral":
(2) Extrinsic Evidence: Language FRE 613(b): “Extrinsic evidence of a prior b) Waiver: a privilege can be waived by a failure to object (raise privilege) or by consent (a) The rule applies only when the secondary evidence is offered to prove the
ii) To a clergy member
inconsistent statement by a witness is not admissible unless the witness is c) Rationale for privilege: iii) The communication was confidential contents of an original writing. It does not apply when the "writing" itself is "not
afforded an opportunity to explain or deny the same and the opposite party is i) Encourage the exchange of info between people in certain relationships, by assuring that (1) "Confidential" means outside the presence of third parties, not intended for further closely related to a controlling issue"
afforded an opportunity to interrogate the witness thereon, or the interests of the confidence will not be betrayed. disclosure except to further the religious purpose of its communication (5) Public Records:
justice otherwise require. This provision does not apply to admissions of a party- d) What Privilege Law to Use: FRE does not define specific privileges, only provides a choice of iv) Made while clergy member was acting in his professional capacity (a) FRE 902(4): properly authenticated copy (certified) of a public record may
opponent as defined in Rule 801(d)(2).” law: v) Some jurisdictions require: Clergy member must have a duty to keep those communications be used instead of the original.
(a) Basic Rule: Extrinsic evidence of a prior inconsistent statement by a i) Language FRE 501, General Rule: “Except as otherwise required by the Constitution of secret under the tenets of his or her religion or organization (6) Originals too Voluminous:
witness is not admissible unless: the United States or provided by Act of Congress or in rules prescribed by the Supreme Court b) Rationale: Encourage free exercise of religion and religious counseling (a) if the original is so voluminous that it would be impractical to produce them
(i) (1) Witness is afforded an opportunity to explain or deny the pursuant to statutory authority, the privilege of a witness, person, government, State, or political 7) Parent-child Privilege
subdivision thereof shall be governed by the principles of the common law as they may be
in court, the court may allow a summary, as long as notice is given to the
statement, AND the opposite party is afforded an opportunity to interrogate a) Only 4 states recognize this privilege adverse party and the originals are available for inspection by the adverse
interpreted by the courts of the United States in the light of reason and experience. However, in b) Rationale:
the witness thereon, civil actions and proceedings, with respect to an element of a claim or defense as to which State party.
1. Note: FRE is more liberal than the traditional rule, and therefore i) Not clear that children are any more likely to discuss private matters with their parents if
law supplies the rule of decision, the privilege of a witness, person, government, State, or political privilege exists (7) Inapplicable to proof of ABSENCE of contents
allows a prior inconsistent statement to come in even before the witness subdivision thereof shall be determined in accordance with State law.” (a) The rule is inapplicable to testimony that books or records have been
ii) Parents have a duty to children and a right to report them if in child's best interest
is asked about the statement, as long as the opportunities discussed (1) State Law: If a civil case applying State substantive law, privileges will be determined examined "and found not to contain designated error." US v. Madera
above are given afterwards. 8) News Reporter's Privilege
in accordance with State law. (8) Impracticable to bring to court:
(ii) OR (2) the interests of justice otherwise require. a) Rule: A substantial minority of state have enacted statutes protecting a journalist's right to
(2) Federal Law: If applying federal substantive law, unless some other law says withhold the sources of stories, and some courts have seen the first amendment's "freedom of press" (a) If the writing appears on something impractical to bring to court, the court
(iii) Exception: provision does not apply to admissions of a party-opponent otherwise, privileges shall be governed by the principles of the federal common law as as assuring a reporter some level of protection in gathering news and safeguarding sources. will find the writing to be a chattel and best evidence does not apply.
interpreted by the courts of the US in the light of reason and experience.
under 801(d)(2) i) Statute: If a statute exists, it most likely only grants journalists an immunity from contempt, 2) Doctrine of Completeness FRE 106
(b) Not remembering: Not remembering making prior statement is an 3) Attorney-Client Privilege and even then, that immunity may be overcome if the interest in protecting sources of news is a) If a party seeks to introduce only part of a document or recorded statement (e.g.,
inconsistent statement a) Elements: outweighed by the need for the testimony. deposition), the other party may require the introduction at the same time of any other part
(3) Foundation: Under the FRE, there is no requirement to lay a foundation i) Attorney - someone who is licensed to practice law in a jurisdiction, or someone who the ii) Constitutional Protection: While some court recognized a qualified privilege of a reporter "which ought in fairness to be considered contemporaneously with it."
before impeaching a witness regarding prior inconsistent acts or statements (i.e., client reasonably believes is licensed to practice law in a jurisdiction to safeguard sources, it appears that a journalist must appear in response to subpoenas issued 3) Authentication
asking about details of time, place, circumstances) ii) Client - someone or institution seeking legal advice by a grand jury a) Language FRE 901: "The requirement of authentication or identification as a
e) Rehabilitation of Witnesses iii) Communication - information passed in either direction, whether written or oral, regarding b) Rationale: encourage people to give reporters information for news condition precedent to admissibility is satisfied by evidence sufficient to support a finding
the legal services. 9) Identity of Police Informer Privilege
i) When is it Allowed: A witness may not be rehabilitated until her credibility has
(1) Criminal behavior - the atty-client privilege does not protect communication that
that the matter in question is what its proponent claims."
been attacked, mere contradiction of the testimony is not enough facilitates criminal behavior
a) Rule: To Protect confidential informers, the police are privileged to refuse to disclose the i) Rule: the proponent must offer a foundation of evidence sufficient to support a
(1) The support must meet the attack: Generally, in rehabilitating a witness, the (2) Crime-fraud exception: the atty-client privilege does not protect communication that identity of the informer who gave them the information that led to the arrest and prosecution of the finding that the document is genuine and is what it purports to be.
support must meet the attack. This is because FRE 403 requires that evidence be facilitates the furtherance or planning of fraud or criminal activity. accused. ii) Exceptions: FRE 903: not required when the genuineness of the document is
excluded when its probative value is not sufficient to justify the confusion and (a) US v. Zolin - If a party makes a preliminary showing of good faith reason to believe b) Rationale: This will encourage citizens to come forward with information concerning the admitted in the pleadings or by other evidence, or if the adverse party fails to raise a
consumption of time that would result from receiving it. that the communication was made in furtherance of a crime or fraud, the trial court has commission of crimes. timely objection to lack of foundation.
(a) E.g., character evidence met with character evidence the authority to examine the contents of the communication in camera (privately) to c) Waiver: privilege is waived once the informer's identity has been revealed
(1) Role of judge/jury: The judge must first determine, considering admissible
determine whether it was made in furtherance of a crime or fraud. d) Exceptions:
(b) Exception: Prior inconsistent statements (see below) evidence, whether a certain, low, threshold of genuineness has been met, then it is
iv) Confidential communication - both the attorney and the client reasonably believe that their i) Material witness: Whenever it appears that the informer may be a material witness on the
up to the jury to decide the genuineness of the document is.
ii) Allowable Methods: communication is not being overheard by outsiders issue of the defendant's guilt, the informer's identity must be disclosed (or the charges against the
(1) Disproving impeaching evidence: the party supporting a witness is (1) Corporations: While the attorney-client privilege belongs to the corporation, rather than defendant must be dismissed). (Roviaro v. U.S) (a) Note: Look to 104(b) for General rule for preliminary fact determinations
permitted to show that evidence offered to impeach is untrue. individual employees, the Scotus in Upjohn rejected the "control group test," thus the (1) Material - reasonable possibility that the informer might have evidence that could iii) Authenticity can be proven by: (specifically stated in FRE as not exhaustive
(a) E.g., an impeachment by an alleged prior inconsistent statement can be privilege extends to lower level employees outside the corporation if (1) the purpose of the exonerate the defendant. list):
rehabilitated by disproving that the statement was even made. communication is to assist or obtain legal advice pertaining to the client and employees are (2) Balancing Test: D's rights in preparing defense v. public interest in protecting law (1) Direct evidence -
(2) Explain the Impeaching Evidence or Misconduct: aware of the purpose, OR (2) employees are communicating with attorney at the direction of enforcement capabilities. (a) Testimony of a witness with knowledge
(a) Evidence: Impeached witnesses will be permitted to explain prior the employees supervisor, OR (3) comm. is within the scope of the employees duty to act on (b) Opinion testimony as to handwriting identification (by any person familiar
behalf of the corporation.
ii) Distinguish: Merely lead to probable cause: No need to disclose informant's ID if only going
inconsistent statements and specific acts of misconduct (no extrinsic evidence to the probable cause for the arrest -- not whether D is guilty. (McCray v. Illinois) with the handwriting, expert testimony, or comparison with some admitedly
(2) 3rd party agents: the attorney-client privilege may extend to communications genuine document),
allowed). 10) Fifth Amendment Privileges
(i) Under 613(b), a party is required to make sure that the witness is 4) Physician-Patient Privilege/Therapist Patient Privilege (c) Voice identification
a) Constitutional Language: "No person…shall be compelled in any criminal case to be a
afforded an opportunity to "explain or deny" the statement. a) Rule: a patient, whether or not a party to the action, has a privilege to refuse to disclose and to witness against himself…" (2) Circumstantial Evidence -
(b) Misconduct: If the witness has been impeached by cross-examination prevent his physician from disclosing, any information acquired by the physician if: (a) Authentication by content( where there are distinctive characteristics
i) Patient consults a doctor for medical treatment
b) Rationale: Unfair to force a person to choose between perjury (i.e. lie) and self-incrimination
about specific acts of misconduct, the witness is always entitled to explain the (i.e. if they tell the truth) and contempt of court (i.e. if they refuse to say anything) linking it to the person),
conduct, however, extrinsic evidence is inadmissible to explain or justify such (1) A court-appointed psychiatrist where treatment is not sought is insufficient c) Rules regarding all Witnesses: (b) Reply letter doctrine (authenticated by evidence that it was received in
(2) Privilege extends to licensed social workers i) The privilege allows a person the right not to answer a question only if the true answer
conduct. response to a communication sent to the claimed author),
(a) Rationale: the work of a social worker depends on confidence and trust, serves the would tend to incriminate him or her of a crime for which he or she can still be punished (c) Process - proving document produced by process which is reliable to
(3) Rehabilitation with Character Evidence public interest by ensuring those who need therapy will get it; 50 state adopted the (1) Thus, if the statute of limitations has run or a person is given immunity, then they
(a) When permitted: Only after a witness's character has been attacked may authenticate,
privilege; if the privilege were not to extend, it would deny the poor ability to keep cannot incoke the privilege
evidence of the witness's good character for truthfulness be introduced. conversations in confidence ii) The privilege applies to:
(d) Ancient Documents Doctrine - evidence that a document, in any form:
(i) After misconduct impeachment: after the impeaching party introduces ii) Information was acquired by the physician while attending the patient (1) every person who gives testimony (i) Is in such condition as to create no suspicion concerning its
evidence that the witness has been convicted of a crime or by asking the (1) Privilege is not restricted to communication, also covers other info acquired by the (2) in every context in which the government has the power to subpoena a person to attend authenticity
witness about specific instances of misconduct, the party supporting the doctor necessary to treat a proceeding as a witness and to hold him or her in contempt of court if he or she refuses (ii) Was in place where it, if authentic, would likely be
witness is entitled to respond with evidence of the witness's good character
(2) Statements to doctor not pertaining to treatment are not covered by the privilege d) Rules regarding the Accused - When an accused is taken into custody or deprived of (iii) AND has been in existence 20 years or more at the time it is offered
for truthfulness. iii) Information is "confidential": freedom, the accused has a right to refuse to respond to questions. (e) Public records - that it is authorized to be recorded and was recorded, and
(1) Obtained outside the presence of third parties, under circumstances indicated that it i) Scope: Only covers testimonial evidence, thus it does not encompass non-testimonial came from public office where items of this nature are kept
(ii) After bias impeachment: merely showing that a witness is biased is not was meant to be kept secret
in itself an attack on the witness's character for truthfulness, therefore the evidence, such as blood samples or routine information iv) Self Authenticating Documents:
b) Waiver: ii) Waiver: (1) Certain kinds of documents or records require no independent proof of
evidence of bias must throw an evil light upon the witness's character for i) Privilege is waived when party puts their mental/physical condition into controversy, (1) Once the accused speaks, that speech can be used against them in a later stage of the
truthfulness before the witness is entitled to support his credibility with (1) E.g., Personal injury suits authenticity, because their nature is prima facie authenticity. The burden then
case. However, this merely constitutes a waiver to that speech, and does not mean that they shifts to adverse party to prove otherwise.
character evidence. ii) patient or physician testifying, waive it entirely.
(iii) After Prior Inconsistent Statement impeachment: Evidence that the iii) by contract, (2) If a person testifies on direct, they waive the privilege of any testimony during the scope (a) Official documents under seal, or certified under seal that signer has
witness made a prior inconsistent statement usually makes an implicit or
iv) by making disclosures to 3rd persons, of their direct examination official capacity and that the signature is genuine 902(1,2)
v) or when the party fails to assert the privilege (b) Notarized documents 902(8)
explicit attack on the truthfulness of the witness, and a witness's credibility
attacked in this fashion may normally be supported with evidence of good
c) Exceptions: ✔ Writings (c) Certified copies of public records 902(4)
i) Dangerous patient exception: several courts have held that when the patient confides an
character for truthfulness intent to harm a third person, the danger of violence may justify the therapist in warning the third
(d) Certified copies of business records of regularly conducted activity 902
(b) Only opinion and reputation evidence allowable: 1) Best Evidence Rule (11,12)
person of the threat. Failure to do so may render the therapist civilly liable for any harm inflicted
(i) FRE 608(a) allows both reputation and opinion evidence to be offered by the patient. a) Language FRE 1002: "To prove the contents of a writing, recording, or photograph, (e) Commercial paper 902(9)
to support the credibility of a witness, but bars extrinsic evidence of specific ii) Criminal cases - some jurisdictions don't allow the privilege in criminal cases the original writing, recording, or photograph is required, except as otherwise provided in (f) Official publications of public authority 902(5)
acts. The rationale is that allowing extrinsic evidence of specific acts would iii) Court ordered mental examinations - not privileged these rules or by Act of Congress." (g) Newspapers and periodicals 902(6)
involve the courts in time consuming mini-trials wasting time. iv) Competency, guardianship, commitment proceedings b) Generally: The best evidence rule requires that, in certain situations, an original or (h) Trade inscriptions and the like affixed in the course of business and
(4) Rehabilitation with Prior Consistent Statements v) Malpractice cases - physician must be able to rebut the evidence against him by patient permissible duplicate of a writing must be presented. indicating ownership or origin 902(7)
(a) The fact that a witness's credibility has been impeached does not vi) When services were sought to assist in planning or committing crime i) Note: does not require the party to offer the best evidence available.
automatically open the door to supporting the credibility by introducing d) Rationale: preclude humiliation, encourage full communication with doctor to receive best c) Rationale: ✔ Opinion Evidence and Expert
evidence of the witness's prior consistent statements. i)
(i) After attack for bias or interest: 5)
treatment
Marital Privilege – Two Types meaning.
Slight differences: in written words or symbols may make a vast difference in
Witness
1. Under FRE 801(d)(1)(B), after a witness has been impeached with a) NONCONFIDENTIAL COMMUNICATION: Spouse-Witness Privilege - ii) Prevents fraud or mistake: Production of the original prevents fraud and mistakes
evidence of bias or interest, only prior consistent statements that were i) Rule: The Spouse-witness privilege, which allows the spouse-witness to refuse to give that might occur if oral testimony or copies were used in lieu thereof. 1) Opinion Definition: an inference or conclusion drawn from facts observed
made prior to the time that the bias or interest arose are admissible to testimony or to exclude any record of the witness-spouse's compelled testimony given in any d) Is the writing at issue an Original?: Under certain circumstances, to be discussed 2) Opinions of Non-Experts
rehabilitate the witness. other unprivileged proceeding, regarding any subject, may be asserted if: next, an original is required. a) Language FRE 701: “If the witness is not testifying as an expert, the witness’
(ii) After attack with inconsistent statements: (1) Couple is validly married at the time of the proceedings, i) "Original": FRE 1001(3) testimony in the form of opinions or inferences is limited to those opinions or inferences
1. A witness's credibility may be rehabilitated by admitting a prior (2) Couple was validly married during the time the communication was made (1) The writing or recording itself, or any counterpart intended to have the same which are
consistent statement after the witness is impeached with a prior (3) Communication was between the spouses
(4) Witness-spouse asserts the privilege effect by a person executing or issuing it. Includes a negative or print, a printout or i) Rationally based on the perception of the witness,
inconsistent statement, if (1) the impeached witness denies making the other readable output that reflects the data accurately. ii) Helpful to a clear understanding of the witness’ testimony or the determination of
inconsistent statement, or (2) the consistent statement helps explain the (a) Because he/she "Holds the privilege"
(5) In Federal courts, the case must be a criminal case ii) "Duplicate" FRE 1001(4) a fact in issue, and
inconsistent statement (unless the witness admits making a flatly
ii) Important Distinctions: applies whether or not the communication was confidential, Allows (1) Defined: A duplicate includes a carbon copy, photostatic copy, microfilm (1) Clarification:
inconsistent statement). (a) Meaning something about which normal persons commonly form opinions,
"unilateral testimony," meaning the other spouse can't force the witness spouse not to testify reproduction, etc.
(iii) After attack on witness's character: generally inadmissible for waste of like size, speed, smell, sound, value, dimensions
iii) Rationale: Want to protect the marriage relationship (2) Admissibility: A duplicate is admissible the same as the original, unless:
time due to not meeting the attack (b) And that testimony which is the clearest, most understandable way of
iv) Exceptions: crimes against the person or property of the other spouse, or cases involving (a) The authenticity of the original is genuinely disputed
(iv) After attack on witness's memory: getting the matter to the jury (like drunk, rather than the details about speech,
crimes against the children of either spouse, civil action or proceeding instituted by one spouse (b) It would be unfair under the circumstances to admit the duplicate in lieu of
1. A prior consistent statement is normally admissible to an breath, etc.)
against the other (divorce), proceedings to determine the mental compentency of either spouse, the original (as where only part of the original was reproduced and the rest is
impeachment on the witness's memory, if the prior consistent Juvenile proceedings involving their children. iii) AND Not based on scientific, technical, or other specialized knowledge within the
necessary for cross examination)
statements was made earlier in time when the witness's memory would v) Waiver: if a specific objection is not made by the spouse entitled to assert the privilege, it is scope of Rule 702.”
have been fresher. e) If not an Original, is an Original required? The best evidence rule requires that an
waived. "original" is required if: b) Rationale: factual conclusions that are within the comprehension of the average lay
(v) Prior Identification of accused: b) CONFIDENTIAL COMMUNICATION: Marital Communications Privilege i) First, Object in question is a "Writing, recording or photograph": FRE 1001 person, then there is no need for the witness to inject their own conclusions, and should
1. Under FRE 801(d)(1)(C), a prior identification of an accused is
i) Rule: the Marital Communications Privilege, which allows either spouse to refuse to (1) Defined very broadly. Includes printed material, cds, movies, x-rays. be left to the jury.
nonhearsay. Usually, however, the evidence is admitted if offered to disclose, or prevent the other spouse from disclosing communications, is established if:
bolster the witness's courtroom identification, and the modern trend (2) Photographs: obscene photo content would be at issue so must have 3) Opinion Testimony by Expert Witness
a) Language FRE 702: "If scientific, technical, or other specialized knowledge will
assist the trier of fact to understand the evidence or determine a fact in issue, a witness
qualified as an expert by knowledge, skill, experience, training, or education, may testify
thereto in the form of an opinion or otherwise, if:
i) the testimony is based upon sufficient facts or data
ii) the testimony is the product of reliable principals and methods, and
iii) the witness has applied the principles and methods reliably to the facts of the
case."
iv) Broken down in to Elements: a witness may testify thereto in the form of an
opinion or otherwise, if
(1) Scientific, technical, or other specialized knowledge will assist the trier of
fact to understand the evidence or determine a fact in issue
(a) Meaning the subject of the testimony relates to materal that is beyond the
common experience of the trier of fact
(2) Witness is qualified as an expert by knowledge, skill, experience, training, or
education
(a) Note: formal education isn't required
(3) The testimony is the product of reliable principals and methods, which are
reliably applied to the facts of the case.
(a) Daubert clarification- the reasoning or methodology underlying the
testimony must be scientifically valid and applicable to the case at hand,
considering the following suggested factors:
(i) Can the expert's theory or technique be tested, and if so, has it been
tested?
(ii) Has the expert's theory or technique been published in journals
subject to peer review?
(iii) With regards to a particular technique, what is its known or potential
rate of error, are there standards for controlling the technique's operation?
If so, were these used in developing the expert's testimony?
(iv) Has the expert's theory or technique achieved general acceptance
with a relevant community?
(v) Whether the experts are testifying about matters stemming directly
from research without regard to litigation or whether theories developed
solely for litigation
(vi) Whether expert is making a big leap in making a conclusion based on
the data in front of them
(vii) Whether the expert has adequately accounted for alternative
explanations
(b) Polygraph tests - split:
(i) Allowable: US v. Piccionna: 11th circ. - No per se rule against the
admission of polygraph evidence - they can be admitted in certain
circumstances:
1. Stipulation by the other side before the test is given
2. Can be used for impeachment or to corroborate the testimony of a
witness if (1) notice, (2) allowing the other side to give its own test, (3)
use of the testimony comports with the FRE
(ii) Unallowable: - state v. porter - Polygraph tests are not sufficiently
reliable, because people are able to train themselves to avoid a bad result
on the polygraph tests
1. SCOTUS in US v. Scheffer - It is not a violation of the
constitutional rights of the defendant to not have a per se rule against
the admissibility of polygraph testimony.
(c) Sobriety tests: US v. Horn - tests can be allowed in as circumstantial
evidence of the officers lay opinion as to the sobriety of the person, but not to
prove actual blood alcohol level
(i) Explanation - this fails the first two prongs of Daubert (no real scientific
basis)
(d) Testimony or reliability of Witness Identification: State v. Chapple -
Expert testimony allowable here, because it is not testifying to the truthfulness,
only to the accuracy that jury could use in applying to the weight of the witness'
testimony
b) Expert Doesn’t Have to First Testify Facts - Language FRE 705: "The expert may
testify in terms of opinion or inference and give reasons therefore without first testifying to
the underlying facts or data, unless the court requires otherwise. The expert may in any
event be required to disclose the underlying facts or data on cross-examination.
i) Broken Down: The expert is not required to first explain the underlying facts or
data, but it may be required on cross examination.
c) What May an Expert Base his Opinion On? - Language FRE 703: “The facts or
data in the particular case upon which an expert bases an opinion or inference may be
those perceived by or made known to the expert at or before the hearing. If of a type
reasonably relied upon by experts in the particular field in forming opinions or inferences
upon the subject, the facts or data need not be admissible in evidence in order for the
opinion or inference to be admitted. Facts or data that are otherwise inadmissible shall not
be disclosed to the jury by the proponent of the opinion or inference unless the court
determines that their probative value in assisting the jury to evaluate the expert’s opinion
substantially outweighs their prejudicial effect.”
i) Broken down into rules:
(1) Expert may base opinion on: The facts or data in the particular case upon
which an expert bases an opinion or inference may be those which are both (1)
perceived by or made known to the expert at or before the hearing, and (2) of a
type reasonably relied upon by experts in the particular field in forming opinions or
inferences upon the subject,
(2) Inadmissible facts opinion is based on: An expert is not precluded from
basing an opinion or inference on those facts which are inadmissible in evidence,
but those inadmissible facts shall not be disclosed to the jury, unless their
probative value in assisting the jury to evaluate the expert's opinion substantially
outweigh their prejudicial effect.
d) What May Expert Testify To? - Language FRE 704:
i) Section(a): Except as provided in subdivision (b), testimony in the form of an
opinion or inference otherwise admissible is not objectionable because it embraces an
ultimate issue to be decided by the trier of fact”
ii) Section (b): No Expert witness testifying with respect to the mental state or
condition of a defendant in a criminal case may state an opinion or inference as to
whether the defendant did or did not have the mental state or condition constituting an
element of the crime charged or of a defense thereto. Such ultimate issues are
matters for the trier or fact alone.
iii) Section (b) Broken down into rules:
(1) Expert's opinion or inference may embrace an ultimate issue to be decided by
the trier of fact, except:
(a) Expert may not testify in legal conclusions or legal terms of art of the
statute defendant is being tried for
(b) Expert cannot testify based on personal assessment of witness' credibility
(c) Expert can respond to hypos that assume that certain facts are true
(i) But cannot be based on facts against the facts in evidence or
imaginary facts - Ingram v. McCuiston
(d) Expert cannot testify to ultimate mental state of a criminal, including
responding to hypothetical questions that mirror the fact pattern in a case (FRE
704(b))
(i) However, this doesn't mean the expert can't testify that a fact similar to
those in evidence are consistent with the conduct of the hypothetical person
suffering from the same mental condition
(ii) “Such ultimate issues are matters for the trier of fact alone.”

Das könnte Ihnen auch gefallen